Difference between revisions of "AoPS Wiki talk:Problem of the Day/September 10, 2011"

(Created page with "Let <cmath>S=\frac{F_1}{3}+\frac{F_2}{9}+\frac{F_3}{27}+\frac{F_4}{81}+\frac{F_5}{243}+\cdots</cmath> Multiplying through by 1/3, we see <cmath>\frac{1}{3}S=\frac{F_1}{9}+\frac{F...")
 
m
 
Line 1: Line 1:
 +
==Solution==
 
Let
 
Let
 
<cmath>S=\frac{F_1}{3}+\frac{F_2}{9}+\frac{F_3}{27}+\frac{F_4}{81}+\frac{F_5}{243}+\cdots</cmath>
 
<cmath>S=\frac{F_1}{3}+\frac{F_2}{9}+\frac{F_3}{27}+\frac{F_4}{81}+\frac{F_5}{243}+\cdots</cmath>

Latest revision as of 08:24, 10 September 2011

Solution

Let \[S=\frac{F_1}{3}+\frac{F_2}{9}+\frac{F_3}{27}+\frac{F_4}{81}+\frac{F_5}{243}+\cdots\] Multiplying through by 1/3, we see \[\frac{1}{3}S=\frac{F_1}{9}+\frac{F_2}{27}+\frac{F_3}{81}+\frac{F_4}{243}+\cdots\] and, noting that $F_{n+1}-F_n=F_{n-1}$, we subtract: \[\frac{2}{3}S=\frac{1}{3}+\frac{F_1}{27}+\frac{F_2}{81}+\frac{F_3}{243}+\cdots\] and, multiplying through by 9, we see S in the right side: \[6S=3+\frac{F_1}{3}+\frac{F_2}{9}+\frac{F_3}{27}+\frac{F_4}{81}+\frac{F_5}{243}+\cdots=3+S\] and thus $5S=3$ and $\boxed{S=\frac{3}{5}}$.